Cauchy's Theorem for Abelian GroupsThe existence of a subgroup of an abelian and finite group $G$An attempted proof of Cauchy's theorem for abelian groups using composition series.What is the difference between Cauchy's Theorem and Cauchy's Theorem for Abelian Groups?Subgroups of Abelian Groups, Theorem of Finite Abelian GroupsLet $G$ be a finite abelian group and let $p$ be a prime that divides order of $G$. then $G$ has an element of order $p$every Abelian group is a converse lagrange theorem groupThe proof of Cauchy's theorem for abelian groupsSame order groupsIn this case, why must $p$ divide $|G/N|$?Proof of Cauchy's Theorem for abelian groupsClarification of Proof of Cauchy's theorem for abelian groupsProof of Cauchy's theorem for finite groups in Dummit and FooteAn attempted proof of Cauchy's theorem for abelian groups using composition series.

Is "plugging out" electronic devices an American expression?

Symmetry in quantum mechanics

What are the advantages and disadvantages of running one shots compared to campaigns?

What do you call something that goes against the spirit of the law, but is legal when interpreting the law to the letter?

Re-submission of rejected manuscript without informing co-authors

Doomsday-clock for my fantasy planet

Can a planet have a different gravitational pull depending on its location in orbit around its sun?

How to answer pointed "are you quitting" questioning when I don't want them to suspect

What to wear for invited talk in Canada

Manga about a female worker who got dragged into another world together with this high school girl and she was just told she's not needed anymore

COUNT(*) or MAX(id) - which is faster?

I see my dog run

What is the command to reset a PC without deleting any files

Is it wise to focus on putting odd beats on left when playing double bass drums?

Does bootstrapped regression allow for inference?

Is there a way to make member function NOT callable from constructor?

What is GPS' 19 year rollover and does it present a cybersecurity issue?

Is there a familial term for apples and pears?

How to deal with fear of taking dependencies

Is domain driven design an anti-SQL pattern?

Is there a name of the flying bionic bird?

map list to bin numbers

Is ipsum/ipsa/ipse a third person pronoun, or can it serve other functions?

Domain expired, GoDaddy holds it and is asking more money



Cauchy's Theorem for Abelian Groups


The existence of a subgroup of an abelian and finite group $G$An attempted proof of Cauchy's theorem for abelian groups using composition series.What is the difference between Cauchy's Theorem and Cauchy's Theorem for Abelian Groups?Subgroups of Abelian Groups, Theorem of Finite Abelian GroupsLet $G$ be a finite abelian group and let $p$ be a prime that divides order of $G$. then $G$ has an element of order $p$every Abelian group is a converse lagrange theorem groupThe proof of Cauchy's theorem for abelian groupsSame order groupsIn this case, why must $p$ divide $|G/N|$?Proof of Cauchy's Theorem for abelian groupsClarification of Proof of Cauchy's theorem for abelian groupsProof of Cauchy's theorem for finite groups in Dummit and FooteAn attempted proof of Cauchy's theorem for abelian groups using composition series.













4












$begingroup$


I am trying to verify the following proposition:




Let $G$ be a finite Abelian group and let $p$ be a prime that divides the
order of $G$. Then $G$ has an element of order $p$.




My proof:
By Lagrange's theorem: $x^G=e$. By assumption we have $kp=|G|$ for some prime $p$. So $e=x^G=x^kp$. Thus $|x^k|=p$. $blacksquare$



The book's proof uses induction and cosets -- is this necessary?



For reference, here's the book proof:




Clearly, this statement is true for the case in which $G$ has ­order 2.
We prove the theorem by using the Second Principle of Mathematical Induction on $|G|$. That is, we assume that the statement is true for all Abelian groups with fewer elements than G and use this assumption to show that the statement is true for G as well. Certainly, G has elements of prime order, for if $|x| = m$ and $m = qn$, where $q$ is prime, then $|x^n| = q$. So let $x$ be an element of $G$ of some prime order $q$, say. If $q=p$, we are finished; so assume that $q neq p$. Since every subgroup of an Abelian group is normal, we may construct the factor group $barG = G/langle xrangle$. Then $barG$ is Abelian and $p$ divides $|G|$, since $|barG| = |G|/q$. By induction, then, $G$ has an element — call it $ylangle xrangle$ — of order $p$. Then, $(ylangle xrangle)^p = y^plangle xrangle = langle xrangle$ and therefore $y^p in langle xrangle$. If $y^p = e$, we are done. If not, then $y^p$ has order $q$ and $y^q$ has order $p$. $blacksquare$











share|cite|improve this question











$endgroup$







  • 4




    $begingroup$
    Google "McKay's Proof of Cauchy Theorem". Increidibly simple, short and elegant, and you don't need to do the abelian and the non-abelian cases separately.
    $endgroup$
    – DonAntonio
    Nov 29 '17 at 23:17











  • $begingroup$
    But your "proof" does not seem to use the fact that $p$ is a prime number? So if your proof were correct, we would have shown that for any divisor $d$ of the group order, there exists an element of order $d$ (in the group). But that is false (do you know a counterexample?). So any valid proof must use in some way the fact that the divisor considered is prime.
    $endgroup$
    – Jeppe Stig Nielsen
    Nov 30 '17 at 13:09















4












$begingroup$


I am trying to verify the following proposition:




Let $G$ be a finite Abelian group and let $p$ be a prime that divides the
order of $G$. Then $G$ has an element of order $p$.




My proof:
By Lagrange's theorem: $x^G=e$. By assumption we have $kp=|G|$ for some prime $p$. So $e=x^G=x^kp$. Thus $|x^k|=p$. $blacksquare$



The book's proof uses induction and cosets -- is this necessary?



For reference, here's the book proof:




Clearly, this statement is true for the case in which $G$ has ­order 2.
We prove the theorem by using the Second Principle of Mathematical Induction on $|G|$. That is, we assume that the statement is true for all Abelian groups with fewer elements than G and use this assumption to show that the statement is true for G as well. Certainly, G has elements of prime order, for if $|x| = m$ and $m = qn$, where $q$ is prime, then $|x^n| = q$. So let $x$ be an element of $G$ of some prime order $q$, say. If $q=p$, we are finished; so assume that $q neq p$. Since every subgroup of an Abelian group is normal, we may construct the factor group $barG = G/langle xrangle$. Then $barG$ is Abelian and $p$ divides $|G|$, since $|barG| = |G|/q$. By induction, then, $G$ has an element — call it $ylangle xrangle$ — of order $p$. Then, $(ylangle xrangle)^p = y^plangle xrangle = langle xrangle$ and therefore $y^p in langle xrangle$. If $y^p = e$, we are done. If not, then $y^p$ has order $q$ and $y^q$ has order $p$. $blacksquare$











share|cite|improve this question











$endgroup$







  • 4




    $begingroup$
    Google "McKay's Proof of Cauchy Theorem". Increidibly simple, short and elegant, and you don't need to do the abelian and the non-abelian cases separately.
    $endgroup$
    – DonAntonio
    Nov 29 '17 at 23:17











  • $begingroup$
    But your "proof" does not seem to use the fact that $p$ is a prime number? So if your proof were correct, we would have shown that for any divisor $d$ of the group order, there exists an element of order $d$ (in the group). But that is false (do you know a counterexample?). So any valid proof must use in some way the fact that the divisor considered is prime.
    $endgroup$
    – Jeppe Stig Nielsen
    Nov 30 '17 at 13:09













4












4








4


2



$begingroup$


I am trying to verify the following proposition:




Let $G$ be a finite Abelian group and let $p$ be a prime that divides the
order of $G$. Then $G$ has an element of order $p$.




My proof:
By Lagrange's theorem: $x^G=e$. By assumption we have $kp=|G|$ for some prime $p$. So $e=x^G=x^kp$. Thus $|x^k|=p$. $blacksquare$



The book's proof uses induction and cosets -- is this necessary?



For reference, here's the book proof:




Clearly, this statement is true for the case in which $G$ has ­order 2.
We prove the theorem by using the Second Principle of Mathematical Induction on $|G|$. That is, we assume that the statement is true for all Abelian groups with fewer elements than G and use this assumption to show that the statement is true for G as well. Certainly, G has elements of prime order, for if $|x| = m$ and $m = qn$, where $q$ is prime, then $|x^n| = q$. So let $x$ be an element of $G$ of some prime order $q$, say. If $q=p$, we are finished; so assume that $q neq p$. Since every subgroup of an Abelian group is normal, we may construct the factor group $barG = G/langle xrangle$. Then $barG$ is Abelian and $p$ divides $|G|$, since $|barG| = |G|/q$. By induction, then, $G$ has an element — call it $ylangle xrangle$ — of order $p$. Then, $(ylangle xrangle)^p = y^plangle xrangle = langle xrangle$ and therefore $y^p in langle xrangle$. If $y^p = e$, we are done. If not, then $y^p$ has order $q$ and $y^q$ has order $p$. $blacksquare$











share|cite|improve this question











$endgroup$




I am trying to verify the following proposition:




Let $G$ be a finite Abelian group and let $p$ be a prime that divides the
order of $G$. Then $G$ has an element of order $p$.




My proof:
By Lagrange's theorem: $x^G=e$. By assumption we have $kp=|G|$ for some prime $p$. So $e=x^G=x^kp$. Thus $|x^k|=p$. $blacksquare$



The book's proof uses induction and cosets -- is this necessary?



For reference, here's the book proof:




Clearly, this statement is true for the case in which $G$ has ­order 2.
We prove the theorem by using the Second Principle of Mathematical Induction on $|G|$. That is, we assume that the statement is true for all Abelian groups with fewer elements than G and use this assumption to show that the statement is true for G as well. Certainly, G has elements of prime order, for if $|x| = m$ and $m = qn$, where $q$ is prime, then $|x^n| = q$. So let $x$ be an element of $G$ of some prime order $q$, say. If $q=p$, we are finished; so assume that $q neq p$. Since every subgroup of an Abelian group is normal, we may construct the factor group $barG = G/langle xrangle$. Then $barG$ is Abelian and $p$ divides $|G|$, since $|barG| = |G|/q$. By induction, then, $G$ has an element — call it $ylangle xrangle$ — of order $p$. Then, $(ylangle xrangle)^p = y^plangle xrangle = langle xrangle$ and therefore $y^p in langle xrangle$. If $y^p = e$, we are done. If not, then $y^p$ has order $q$ and $y^q$ has order $p$. $blacksquare$








group-theory proof-verification finite-groups






share|cite|improve this question















share|cite|improve this question













share|cite|improve this question




share|cite|improve this question








edited Nov 29 '17 at 23:24









José Carlos Santos

173k23133241




173k23133241










asked Nov 29 '17 at 23:10









yoshiyoshi

1,256917




1,256917







  • 4




    $begingroup$
    Google "McKay's Proof of Cauchy Theorem". Increidibly simple, short and elegant, and you don't need to do the abelian and the non-abelian cases separately.
    $endgroup$
    – DonAntonio
    Nov 29 '17 at 23:17











  • $begingroup$
    But your "proof" does not seem to use the fact that $p$ is a prime number? So if your proof were correct, we would have shown that for any divisor $d$ of the group order, there exists an element of order $d$ (in the group). But that is false (do you know a counterexample?). So any valid proof must use in some way the fact that the divisor considered is prime.
    $endgroup$
    – Jeppe Stig Nielsen
    Nov 30 '17 at 13:09












  • 4




    $begingroup$
    Google "McKay's Proof of Cauchy Theorem". Increidibly simple, short and elegant, and you don't need to do the abelian and the non-abelian cases separately.
    $endgroup$
    – DonAntonio
    Nov 29 '17 at 23:17











  • $begingroup$
    But your "proof" does not seem to use the fact that $p$ is a prime number? So if your proof were correct, we would have shown that for any divisor $d$ of the group order, there exists an element of order $d$ (in the group). But that is false (do you know a counterexample?). So any valid proof must use in some way the fact that the divisor considered is prime.
    $endgroup$
    – Jeppe Stig Nielsen
    Nov 30 '17 at 13:09







4




4




$begingroup$
Google "McKay's Proof of Cauchy Theorem". Increidibly simple, short and elegant, and you don't need to do the abelian and the non-abelian cases separately.
$endgroup$
– DonAntonio
Nov 29 '17 at 23:17





$begingroup$
Google "McKay's Proof of Cauchy Theorem". Increidibly simple, short and elegant, and you don't need to do the abelian and the non-abelian cases separately.
$endgroup$
– DonAntonio
Nov 29 '17 at 23:17













$begingroup$
But your "proof" does not seem to use the fact that $p$ is a prime number? So if your proof were correct, we would have shown that for any divisor $d$ of the group order, there exists an element of order $d$ (in the group). But that is false (do you know a counterexample?). So any valid proof must use in some way the fact that the divisor considered is prime.
$endgroup$
– Jeppe Stig Nielsen
Nov 30 '17 at 13:09




$begingroup$
But your "proof" does not seem to use the fact that $p$ is a prime number? So if your proof were correct, we would have shown that for any divisor $d$ of the group order, there exists an element of order $d$ (in the group). But that is false (do you know a counterexample?). So any valid proof must use in some way the fact that the divisor considered is prime.
$endgroup$
– Jeppe Stig Nielsen
Nov 30 '17 at 13:09










3 Answers
3






active

oldest

votes


















14












$begingroup$

Your proof is incorrect. You know that $(x^k)^p=e$ (for any $xin G$), but this does not necessarily mean $x^k$ has order $p$. All it tells you is that the order of $x^k$ divides $p$, so it is either $1$ or $p$.



In fact, your approach cannot work without some major modification. It is possible that $x^k=e$ for all $xin G$, so there is no element of the form $x^k$ which has order $p$. For instance, if $G=(mathbbZ/pmathbbZ)^2$, then $|G|=p^2$ so $k=p$, but $x^p=e$ for all $xin G$.






share|cite|improve this answer









$endgroup$








  • 1




    $begingroup$
    (+1) Your answer is better than mine, since you explain why that proof could not work.
    $endgroup$
    – José Carlos Santos
    Nov 30 '17 at 9:51


















6












$begingroup$

From $x^kp=e$, what you can deduce is that $(x^k)^p=e$, and therefore, that $|x^k|$ divides $p$; that is, that it is equal to $p$ or equal to $1$ (that is, $x^k=e$). How do you know that it is equal to $p$?



And where did you use that $G$ is Abelian?






share|cite|improve this answer











$endgroup$




















    0












    $begingroup$

    Here is a simple proof due to Frobenius, I believe.
    Let $x_1, x_2, ..., x_n$ be the elements of the group $G$. Let $r_i$ be the order of the element $x_i$. Furthermore, let $$Z = big( mathbbZ / r_1 mathbbZ, + big) times big( mathbbZ / r_2 mathbbZ, + big) times ldots times big( mathbbZ / r_n mathbbZ, + big)$$
    Now define a map $varphi$ from $Z$ into $G$ by $varphi(k_1, k_2, ldots, k_n) = x_1^k_1 x_2^k_2 dots x_n^k_n$. As $G$ is abelian it is easy to see that $varphi$ is a homomorphism. Moreover $varphi$ is surjective as $varphi(0,0, ldots, 1, ldots, 0) = x_i$. If $K = ker varphi$, then $Z/K cong G$ and therefore $|Z| = |K| cdot |G|$. Since $p$ divides $|G|$, it follows that $p$ divides $|Z|$. But $|Z| = r_1 times r_2 ldots r_n$. Hence one of the $r_i$ is divisible by $p$.






    share|cite|improve this answer









    $endgroup$













      Your Answer





      StackExchange.ifUsing("editor", function ()
      return StackExchange.using("mathjaxEditing", function ()
      StackExchange.MarkdownEditor.creationCallbacks.add(function (editor, postfix)
      StackExchange.mathjaxEditing.prepareWmdForMathJax(editor, postfix, [["$", "$"], ["\\(","\\)"]]);
      );
      );
      , "mathjax-editing");

      StackExchange.ready(function()
      var channelOptions =
      tags: "".split(" "),
      id: "69"
      ;
      initTagRenderer("".split(" "), "".split(" "), channelOptions);

      StackExchange.using("externalEditor", function()
      // Have to fire editor after snippets, if snippets enabled
      if (StackExchange.settings.snippets.snippetsEnabled)
      StackExchange.using("snippets", function()
      createEditor();
      );

      else
      createEditor();

      );

      function createEditor()
      StackExchange.prepareEditor(
      heartbeatType: 'answer',
      autoActivateHeartbeat: false,
      convertImagesToLinks: true,
      noModals: true,
      showLowRepImageUploadWarning: true,
      reputationToPostImages: 10,
      bindNavPrevention: true,
      postfix: "",
      imageUploader:
      brandingHtml: "Powered by u003ca class="icon-imgur-white" href="https://imgur.com/"u003eu003c/au003e",
      contentPolicyHtml: "User contributions licensed under u003ca href="https://creativecommons.org/licenses/by-sa/3.0/"u003ecc by-sa 3.0 with attribution requiredu003c/au003e u003ca href="https://stackoverflow.com/legal/content-policy"u003e(content policy)u003c/au003e",
      allowUrls: true
      ,
      noCode: true, onDemand: true,
      discardSelector: ".discard-answer"
      ,immediatelyShowMarkdownHelp:true
      );



      );













      draft saved

      draft discarded


















      StackExchange.ready(
      function ()
      StackExchange.openid.initPostLogin('.new-post-login', 'https%3a%2f%2fmath.stackexchange.com%2fquestions%2f2543677%2fcauchys-theorem-for-abelian-groups%23new-answer', 'question_page');

      );

      Post as a guest















      Required, but never shown

























      3 Answers
      3






      active

      oldest

      votes








      3 Answers
      3






      active

      oldest

      votes









      active

      oldest

      votes






      active

      oldest

      votes









      14












      $begingroup$

      Your proof is incorrect. You know that $(x^k)^p=e$ (for any $xin G$), but this does not necessarily mean $x^k$ has order $p$. All it tells you is that the order of $x^k$ divides $p$, so it is either $1$ or $p$.



      In fact, your approach cannot work without some major modification. It is possible that $x^k=e$ for all $xin G$, so there is no element of the form $x^k$ which has order $p$. For instance, if $G=(mathbbZ/pmathbbZ)^2$, then $|G|=p^2$ so $k=p$, but $x^p=e$ for all $xin G$.






      share|cite|improve this answer









      $endgroup$








      • 1




        $begingroup$
        (+1) Your answer is better than mine, since you explain why that proof could not work.
        $endgroup$
        – José Carlos Santos
        Nov 30 '17 at 9:51















      14












      $begingroup$

      Your proof is incorrect. You know that $(x^k)^p=e$ (for any $xin G$), but this does not necessarily mean $x^k$ has order $p$. All it tells you is that the order of $x^k$ divides $p$, so it is either $1$ or $p$.



      In fact, your approach cannot work without some major modification. It is possible that $x^k=e$ for all $xin G$, so there is no element of the form $x^k$ which has order $p$. For instance, if $G=(mathbbZ/pmathbbZ)^2$, then $|G|=p^2$ so $k=p$, but $x^p=e$ for all $xin G$.






      share|cite|improve this answer









      $endgroup$








      • 1




        $begingroup$
        (+1) Your answer is better than mine, since you explain why that proof could not work.
        $endgroup$
        – José Carlos Santos
        Nov 30 '17 at 9:51













      14












      14








      14





      $begingroup$

      Your proof is incorrect. You know that $(x^k)^p=e$ (for any $xin G$), but this does not necessarily mean $x^k$ has order $p$. All it tells you is that the order of $x^k$ divides $p$, so it is either $1$ or $p$.



      In fact, your approach cannot work without some major modification. It is possible that $x^k=e$ for all $xin G$, so there is no element of the form $x^k$ which has order $p$. For instance, if $G=(mathbbZ/pmathbbZ)^2$, then $|G|=p^2$ so $k=p$, but $x^p=e$ for all $xin G$.






      share|cite|improve this answer









      $endgroup$



      Your proof is incorrect. You know that $(x^k)^p=e$ (for any $xin G$), but this does not necessarily mean $x^k$ has order $p$. All it tells you is that the order of $x^k$ divides $p$, so it is either $1$ or $p$.



      In fact, your approach cannot work without some major modification. It is possible that $x^k=e$ for all $xin G$, so there is no element of the form $x^k$ which has order $p$. For instance, if $G=(mathbbZ/pmathbbZ)^2$, then $|G|=p^2$ so $k=p$, but $x^p=e$ for all $xin G$.







      share|cite|improve this answer












      share|cite|improve this answer



      share|cite|improve this answer










      answered Nov 29 '17 at 23:14









      Eric WofseyEric Wofsey

      192k14220352




      192k14220352







      • 1




        $begingroup$
        (+1) Your answer is better than mine, since you explain why that proof could not work.
        $endgroup$
        – José Carlos Santos
        Nov 30 '17 at 9:51












      • 1




        $begingroup$
        (+1) Your answer is better than mine, since you explain why that proof could not work.
        $endgroup$
        – José Carlos Santos
        Nov 30 '17 at 9:51







      1




      1




      $begingroup$
      (+1) Your answer is better than mine, since you explain why that proof could not work.
      $endgroup$
      – José Carlos Santos
      Nov 30 '17 at 9:51




      $begingroup$
      (+1) Your answer is better than mine, since you explain why that proof could not work.
      $endgroup$
      – José Carlos Santos
      Nov 30 '17 at 9:51











      6












      $begingroup$

      From $x^kp=e$, what you can deduce is that $(x^k)^p=e$, and therefore, that $|x^k|$ divides $p$; that is, that it is equal to $p$ or equal to $1$ (that is, $x^k=e$). How do you know that it is equal to $p$?



      And where did you use that $G$ is Abelian?






      share|cite|improve this answer











      $endgroup$

















        6












        $begingroup$

        From $x^kp=e$, what you can deduce is that $(x^k)^p=e$, and therefore, that $|x^k|$ divides $p$; that is, that it is equal to $p$ or equal to $1$ (that is, $x^k=e$). How do you know that it is equal to $p$?



        And where did you use that $G$ is Abelian?






        share|cite|improve this answer











        $endgroup$















          6












          6








          6





          $begingroup$

          From $x^kp=e$, what you can deduce is that $(x^k)^p=e$, and therefore, that $|x^k|$ divides $p$; that is, that it is equal to $p$ or equal to $1$ (that is, $x^k=e$). How do you know that it is equal to $p$?



          And where did you use that $G$ is Abelian?






          share|cite|improve this answer











          $endgroup$



          From $x^kp=e$, what you can deduce is that $(x^k)^p=e$, and therefore, that $|x^k|$ divides $p$; that is, that it is equal to $p$ or equal to $1$ (that is, $x^k=e$). How do you know that it is equal to $p$?



          And where did you use that $G$ is Abelian?







          share|cite|improve this answer














          share|cite|improve this answer



          share|cite|improve this answer








          edited Nov 30 '17 at 6:57

























          answered Nov 29 '17 at 23:15









          José Carlos SantosJosé Carlos Santos

          173k23133241




          173k23133241





















              0












              $begingroup$

              Here is a simple proof due to Frobenius, I believe.
              Let $x_1, x_2, ..., x_n$ be the elements of the group $G$. Let $r_i$ be the order of the element $x_i$. Furthermore, let $$Z = big( mathbbZ / r_1 mathbbZ, + big) times big( mathbbZ / r_2 mathbbZ, + big) times ldots times big( mathbbZ / r_n mathbbZ, + big)$$
              Now define a map $varphi$ from $Z$ into $G$ by $varphi(k_1, k_2, ldots, k_n) = x_1^k_1 x_2^k_2 dots x_n^k_n$. As $G$ is abelian it is easy to see that $varphi$ is a homomorphism. Moreover $varphi$ is surjective as $varphi(0,0, ldots, 1, ldots, 0) = x_i$. If $K = ker varphi$, then $Z/K cong G$ and therefore $|Z| = |K| cdot |G|$. Since $p$ divides $|G|$, it follows that $p$ divides $|Z|$. But $|Z| = r_1 times r_2 ldots r_n$. Hence one of the $r_i$ is divisible by $p$.






              share|cite|improve this answer









              $endgroup$

















                0












                $begingroup$

                Here is a simple proof due to Frobenius, I believe.
                Let $x_1, x_2, ..., x_n$ be the elements of the group $G$. Let $r_i$ be the order of the element $x_i$. Furthermore, let $$Z = big( mathbbZ / r_1 mathbbZ, + big) times big( mathbbZ / r_2 mathbbZ, + big) times ldots times big( mathbbZ / r_n mathbbZ, + big)$$
                Now define a map $varphi$ from $Z$ into $G$ by $varphi(k_1, k_2, ldots, k_n) = x_1^k_1 x_2^k_2 dots x_n^k_n$. As $G$ is abelian it is easy to see that $varphi$ is a homomorphism. Moreover $varphi$ is surjective as $varphi(0,0, ldots, 1, ldots, 0) = x_i$. If $K = ker varphi$, then $Z/K cong G$ and therefore $|Z| = |K| cdot |G|$. Since $p$ divides $|G|$, it follows that $p$ divides $|Z|$. But $|Z| = r_1 times r_2 ldots r_n$. Hence one of the $r_i$ is divisible by $p$.






                share|cite|improve this answer









                $endgroup$















                  0












                  0








                  0





                  $begingroup$

                  Here is a simple proof due to Frobenius, I believe.
                  Let $x_1, x_2, ..., x_n$ be the elements of the group $G$. Let $r_i$ be the order of the element $x_i$. Furthermore, let $$Z = big( mathbbZ / r_1 mathbbZ, + big) times big( mathbbZ / r_2 mathbbZ, + big) times ldots times big( mathbbZ / r_n mathbbZ, + big)$$
                  Now define a map $varphi$ from $Z$ into $G$ by $varphi(k_1, k_2, ldots, k_n) = x_1^k_1 x_2^k_2 dots x_n^k_n$. As $G$ is abelian it is easy to see that $varphi$ is a homomorphism. Moreover $varphi$ is surjective as $varphi(0,0, ldots, 1, ldots, 0) = x_i$. If $K = ker varphi$, then $Z/K cong G$ and therefore $|Z| = |K| cdot |G|$. Since $p$ divides $|G|$, it follows that $p$ divides $|Z|$. But $|Z| = r_1 times r_2 ldots r_n$. Hence one of the $r_i$ is divisible by $p$.






                  share|cite|improve this answer









                  $endgroup$



                  Here is a simple proof due to Frobenius, I believe.
                  Let $x_1, x_2, ..., x_n$ be the elements of the group $G$. Let $r_i$ be the order of the element $x_i$. Furthermore, let $$Z = big( mathbbZ / r_1 mathbbZ, + big) times big( mathbbZ / r_2 mathbbZ, + big) times ldots times big( mathbbZ / r_n mathbbZ, + big)$$
                  Now define a map $varphi$ from $Z$ into $G$ by $varphi(k_1, k_2, ldots, k_n) = x_1^k_1 x_2^k_2 dots x_n^k_n$. As $G$ is abelian it is easy to see that $varphi$ is a homomorphism. Moreover $varphi$ is surjective as $varphi(0,0, ldots, 1, ldots, 0) = x_i$. If $K = ker varphi$, then $Z/K cong G$ and therefore $|Z| = |K| cdot |G|$. Since $p$ divides $|G|$, it follows that $p$ divides $|Z|$. But $|Z| = r_1 times r_2 ldots r_n$. Hence one of the $r_i$ is divisible by $p$.







                  share|cite|improve this answer












                  share|cite|improve this answer



                  share|cite|improve this answer










                  answered Nov 30 '17 at 8:27









                  Group NutGroup Nut

                  1




                  1



























                      draft saved

                      draft discarded
















































                      Thanks for contributing an answer to Mathematics Stack Exchange!


                      • Please be sure to answer the question. Provide details and share your research!

                      But avoid


                      • Asking for help, clarification, or responding to other answers.

                      • Making statements based on opinion; back them up with references or personal experience.

                      Use MathJax to format equations. MathJax reference.


                      To learn more, see our tips on writing great answers.




                      draft saved


                      draft discarded














                      StackExchange.ready(
                      function ()
                      StackExchange.openid.initPostLogin('.new-post-login', 'https%3a%2f%2fmath.stackexchange.com%2fquestions%2f2543677%2fcauchys-theorem-for-abelian-groups%23new-answer', 'question_page');

                      );

                      Post as a guest















                      Required, but never shown





















































                      Required, but never shown














                      Required, but never shown












                      Required, but never shown







                      Required, but never shown

































                      Required, but never shown














                      Required, but never shown












                      Required, but never shown







                      Required, but never shown







                      Popular posts from this blog

                      Triangular numbers and gcdProving sum of a set is $0 pmod n$ if $n$ is odd, or $fracn2 pmod n$ if $n$ is even?Is greatest common divisor of two numbers really their smallest linear combination?GCD, LCM RelationshipProve a set of nonnegative integers with greatest common divisor 1 and closed under addition has all but finite many nonnegative integers.all pairs of a and b in an equation containing gcdTriangular Numbers Modulo $k$ - Hit All Values?Understanding the Existence and Uniqueness of the GCDGCD and LCM with logical symbolsThe greatest common divisor of two positive integers less than 100 is equal to 3. Their least common multiple is twelve times one of the integers.Suppose that for all integers $x$, $x|a$ and $x|b$ if and only if $x|c$. Then $c = gcd(a,b)$Which is the gcd of 2 numbers which are multiplied and the result is 600000?

                      Ingelân Ynhâld Etymology | Geografy | Skiednis | Polityk en bestjoer | Ekonomy | Demografy | Kultuer | Klimaat | Sjoch ek | Keppelings om utens | Boarnen, noaten en referinsjes Navigaasjemenuwww.gov.ukOffisjele webside fan it regear fan it Feriene KeninkrykOffisjele webside fan it Britske FerkearsburoNederlânsktalige ynformaasje fan it Britske FerkearsburoOffisjele webside fan English Heritage, de organisaasje dy't him ynset foar it behâld fan it Ingelske kultuergoedYnwennertallen fan alle Britske stêden út 'e folkstelling fan 2011Notes en References, op dizze sideEngland

                      Հադիս Բովանդակություն Անվանում և նշանակություն | Դասակարգում | Աղբյուրներ | Նավարկման ցանկ